GMAT Critical Reasoning

Home > GMAT Test > GMAT Critical Reasoning Questions

Next steps

Source: PREP

Level: 5

The proposal to hire ten new police officers in Middletown is quite foolish. There is sufficient funding to pay the salaries of the new officers, but not the salaries of additional court and prison employees to process the increased caseload of arrests and convictions that new officers usually generate.

Which of the following, if true, will most seriously weaken the conclusion drawn above?

  • A Studies had shown that an increase in city's place force does not necessarily reduce crime
  • B When one major city increased its police force by 19% last year, there were 40% more arrests and 13% more convictions
  • C If funding for the new police officers' salaries is approved, support for other city services will have to be reduced during the next fiscal year
  • D In most US cities, not all arrests result in convictions, and not all convictions result in prison terms
  • E Middletown's ratio of police officers to citizens has reached a level at which an increase in the number of officers will have a deterrent effect on crime

Show Answer

Previous       Next